The years in which each of the monuments were begun can be completely determined if which one of the following is dis...

yuetngan on July 27, 2020

WHy can't we say answer choice C is correct?

I understand how answer choice E could work, but answer choice C could also work if F and H are interchangeable? Please explain.

Replies
Create a free account to read and take part in forum discussions.

Already have an account? log in

yuetngan on July 27, 2020

Sorry, I meant S and H are interchangeable, not F. Why isn't answer choice C also correct? Which answer choice to choose?

Skylar on July 27, 2020

@yuetngan, happy to help!

We are asked to identify which answer choice will fully determine the arrangement. (E) is the correct answer.

(E) "S was begun in 604."
__
__ __ __ S __
1 2 3 4 5

Rule #2 tells us that H was begun no earlier than 604, so it was either begun in 604 or 605. Now that we have S occupying 604, H cannot go there and the only option left for H is 605.
__
__ __ __ S H
1 2 3 4 5

Rule #1 tells us that G began in an earlier year than L, which began in an earlier year than F. Therefore, we need to place G, L, and F in different years in that order. Since we only have three different years left open, there is only one way to arrange this.
__
G L F S H
1 2 3 4 5

M is the only variable left, so we assign it to the only open spot- 601. Rule #1 tells us that M was begun in either 601, 602, or 603, so this is valid.
M
G L F S H
1 2 3 4 5

Our arrangement is fully determined and (E) is correct. Now, let's look at why (C) is incorrect.

(C) "H was begun in 605."
__
__ __ __ __ H
1 2 3 4 5

There are no deductions we can make from having H in 605. We know from Rule #3 that we will have M at some point within the first three years, but we don't have enough information to know which year specifically. Similarly, we know from Rule #1 that we will have the order G-L-F, but there are different ways to make this fit and we don't have enough information to choose one of the multiple possibilities. Lastly, we have no restrictions on S, so we have no direction on where to place it. Therefore, we could create various scenarios from this starting point of H in 605. This means the arrangement is not fully determined and (C) is incorrect.

Does that make sense? Please let us know if you have any other questions!